What is the value of n?

4/5 < n/7 < 8/9
( n has to be greater than 4 but less than 8 )​

Answers

Answer 1

Answer:

n = 6

Step-by-step explanation:

If n/7 < 8/9, then n/7 < 1

n/7 < 1

n < 7

That leaves us with 5 and 6. With this, we use trial and error.

4/5 > 5/7 (not we want)

4/5 < 6/7 (what we want)

The answer is 6/7 or n = 6


Related Questions

Simplify 7a – 11b + 4ab – 6a + 5b.

Answers

7a-6a-11b+5b+4ab
=a-6b+4ab
Final answer

Using the Time Series Data for the US Nuclear Electric Power Production, calculate the exponential smoothing forecasts for 2005 through 2010 using alpha equals 0.2. Find the forecast error for each time period. If there is n

Answers

Answer:

????????????

I don't know

Perimeter (numerical)

Answers

Answer:

270 m

Step-by-step explanation:

Add up all the sides

P = 19 +18.8+18.8 +18.8+18.8+40.8+19+40.8+18.8+18.8+18.8+18.8

P = 270 m

find the constant of variation k for the direct variation. 4x = -y

Answers

Answer:

k = - 4

Step-by-step explanation:

The equation representing direct variation is

y = kx ← k is the constant of variation

Given

4x = - y ( multiply both sides by - 1 )

- 4x = y , that is

y = - 4x ← in standard form

with k = - 4

construct an angle that bisect 120°​

Answers

Answer:

just make a 120 angle and divide it by 2, 60.

Which graph represents y = |xl?
A
B
C
D

Answers

Answer:

B

Step-by-step explanation:

The equation represented in the question is the parent absolute value question. If you know the different parent functions, then the answer is obvious because absolute value equations always form a V. However, if you do not do this then you can create a table and plugin values. Plugin numbers like 0, -1, and 1 for X and solve for Y. Finally, graph these points and see what graph best fits. If needed you can also plug in more points.

Answer:

B.

Step-by-step explanation:

I got it correct on the warm up

The director of special events for Sun City believed that the amount of money spent on fireworks displays on the 4th of July was predictive of attendance at the Fall Festival held in October. She gathered the following data to test her suspicion.

4th of July ($000) Fall Festival (000)
10.6  8.8
 8.5  6.4
12.5 10.8
 9.0 10.2
 5.5   6.0
12 11.1
 8.0   7.5
 7.5   8.4
 9.0   9.5
10   9.8
 7.5   6.6
10 10.1
 6.0   6.1
12 11.3
10.5   8.8

Required:
Determine the regression equation. Is the amount spent on fireworks related to attendance at the Fall Festival? Conduct a hypothesis test to determine if there is a problem with autocorrelation.

Answers

Answer:

The complete question and its solution in the attached file please find it.

Step-by-step explanation:

Solve this equation for x. Round your answer to the nearest hundredth. 0.77 = log x​

Answers

Answer:

x ≈ 5.89

General Formulas and Concepts:

Pre-Algebra

Equality Properties

Algebra II

Exponential to Logarithmic:                                                                             [tex]\displaystyle b^m=x \rightarrow log_bx=m[/tex]

Step-by-step explanation:

Step 1: Define

Identify

0.77 = log(x)

Step 2: Solve for x

[Equality Property] Raise both sides to the 10th power:                                [tex]\displaysytle 10^{0.77} = 10^{logx}[/tex]Simplify:                                                                                                             [tex]\displaysytle x = 10^{0.77}[/tex]Evaluate:                                                                                                           [tex]\displaysytle x = 5.88844[/tex]

A trough has ends shaped like isosceles triangles, with width 2 m and height 5 m, and the trough is 18 m long. Water is being pumped into the trough at a rate of 8 m3/min. At what rate (in m/min) does the height of the water change when the water is 2 m deep

Answers

9514 1404 393

Answer:

   5/9 m/min

Step-by-step explanation:

The depth of the water is 2/5 of the depth of the trough, so the width of the surface will be 2/5 of the width of the trough:

  2/5 × 2 m = 4/5 m

Then the surface area of the water is ...

  A = LW = (18 m)(4/5 m) = 14.4 m²

The rate of change of height multiplied by the area gives the rate of change of volume:

  8 m³/min = (14.4 m²)(h')

  h' = (8 m³/min)/(14.4 m²) = 5/9 m/min

Find the slope of the line #67

Answers

Vas happenin
Hope your day is going well
The slope form is y2-y1/x2-x1
So I’m going to use (1,1) (-1,4)
Y2= 4
Y1=1
X2= -1
X1= 1
Then you plug in those equations 4-1/ -1 -1
3/-2 that’s the fraction form
The decimal form is -1.5 this is the slope
Hope this helps *smiles*

The wholesale price of 6 oz plastic bottles is 6 cents how many plastic bottles can be purchased for $98.41

Answers

Answer:

1640

Step-by-step explanation:

Take the total amount and divide by the amount for one

Make sure to write 6 cent in dollar form (.06)

98.41 / .06

1640.1666

Round down since we need to buy whole bottles

1640

Find the medien: 16,12,10,15,7,9,16

Answers

Answer:

12

Step-by-step explanation:

arrange the numbers in ascending order and cross out from either side till you have a middle line

FINAL ANSWER:

12

Step-by-step explanation:

Median is the middle number in the data set.

so first of ... we need to arrange the group of numbers from lower to greater.

16, 12, 10, 15, 7, 9, 16 ⇒ 7, 9, 10, 12, 15, 16, 16

Now that we have arranged the numbers from least to greatest all we need to do is to find the middle number of the data set (data set? they are the group of numbers)

Ok, so what you want to do here is to just count the numbers until you get to the middle number of the data set...

7, 9, 10, 12, 15, 16, 16

the median in the given data set is 12.

I hope this helps you!!! Let me know if my answer is incorrect or not...

HAVE A GREAT DAY AND GOD BLESS YOU ;)!!!

Britany wants to read a book. In her room, she has 5 mysteries, 15 historical fictions, 12 modern fantasies, and 7 blographies.
How many different choices are available?


pleaseee

Answers

Answer:

39

Step-by-step explanation:

5 + 15 + 12 + 7 = 39

Answer:

39

Step-by-step explanation:

5 + 15 = 20, 12 + 7 = 19, 20 + 19 = 39.

Simplify: 3.5 x 10^-2 + 2.3 x 10^-2

Answers

Given:

The given expression is:

[tex]3.5\times 10^{-2}+2.3\times 10^{-2}[/tex]

To find:

The simplified form of the given expression.

Solution:

We have,

[tex]3.5\times 10^{-2}+2.3\times 10^{-2}[/tex]

It can be written as:

[tex]=(3.5+2.3)\times 10^{-2}[/tex]

[tex]=5.8\times 10^{-2}[/tex]

Therefore, the simplified form of the given expression is [tex]5.8\times 10^{-2}[/tex].

Michael drove 210 miles in 3 1/2. Jordan drove 330 miles in 6 hours. Which is an accurate comparison of the rates at which the two people drove?

Answers

Michael = 210 / 3.5 = 60 miles per hour

Jordan = 330/ 6 =55 miles per hour

Jordan drove 5 miles per hour slower than michael

What is the value of the expression (2x + y) (2x - y) when x = 4 and y = -5?

Answers

Answer:

39

Step-by-step explanation:

1. (2(4)-5)(2(4)+5)

2.(3)(13)

3.39

Answer:

Step-by-step explanation:

This is a difference of squares question. You should 64 = 25 = 39 Let's see if that happens.

Difference of squares

(2x - y) ( 2x + y) = 4x^2 - y^2

4(4)^2 - (5)^2

64 - 25 = 39

Now do the question exactly as it is written.

(2*4 - -5)(2*4 + -5)

(8 +5)(8 - 5)

3 * 13

39

They really do give the same answer.

Find the area of athletic field if it's length is 120cm and its width is 28cm .A. 397.6cm B. 3360cm C. 296 cm D. 4592cm E. 3356cm​

Answers

Answer:

B 3360

Step-by-step explanation:

Area of Rectangle = Length X Width

120 X 28

= 3360 cm

Answered by Gauthmath

Select the correct answer.
Which is the minimum or maximum value of the given function?
dndnsn

Answers

Answer:

See explanation

Step-by-step explanation:

The question is incomplete, as the function is not given. So, I will make an assumption.

A quadratic function is represented as:

[tex]f(x) = ax^2 + bx + c[/tex]

If [tex]a > 0[/tex], then the function has a minimum x value

E.g. [tex]f(x) = 4x^2 - 5x + 8[/tex] ------ [tex]4 > 0[/tex]

Else, then the function has a maximum x value

E.g. [tex]f(x)= -4x^2 -5x + 8[/tex] ---- [tex]-4 < 0[/tex]

The maximum or minimum x value is calculated using:

[tex]x = -\frac{b}{2a}[/tex]

For instance, the maximum of [tex]f(x)= -4x^2 -5x + 8[/tex] is:

[tex]x = -\frac{-5}{2*-4}[/tex]

[tex]x = -\frac{5}{8}[/tex]

So, the maximum of the function is:

[tex]f(x)= -4x^2 -5x + 8[/tex]

[tex]f(-\frac{5}{8}) = -4 * (-\frac{5}{8})^2 - 5 *(-\frac{5}{8}) +8[/tex]

[tex]f(-\frac{5}{8}) = 9.5625[/tex]

Which expression is equal to
(3x – 4)(2x – 5)?

Answers

Answer:

6x^2-23x+20

Step-by-step explanation:

i think the expanded form of that equation is equal to it.

(3x-4)(2x-5)

3x(2x-5)-4(2x-5)

6x^2-15x-8x+20

6x^2-23x+20

I hope this helps and sorry if it's wrong

6x^2-23x+20 sorry if it’s wrong I’m not sure

The line l with equation x - 2y + 2 = 0 crosses the y-axis at the point P. The line
m with equation 3x + y - 15 = 0 crosses the y-axis at the point Q and intersects
l at the point R. Find the area of triangle PQR.​

Answers

Answer:

Area of ΔPQR is 28 units²

Step-by-step explanation:

-P is the point with coordinates ( 0, y-intercept for line x-2y+2 =0)

-rearrange the equation in the point-slope form y=mx+b  to find the y coordinate of the point P( 0, b)

x-2y+2 = 0, subtract x and 2 from both sides

-2y = -x-2, divide by -2 both sides

y= (1/2)x +1 so b=1 and P (0, 1)

-Q  is the point with coordinates ( 0, y-intercept for line 3x+y -15 =0)

-rearrange the equation in the point-slope form y=mx+b  to find the y coordinate of the point Q( 0, b)

3x +y -15 =0, subtract 3x and add 15 to both sides

y= -3x +15 so b=15 and Q(0,15)

-R is the intersection of the two lines so is the solution of the system of equations y= (1/2)x +1 and y= -3x +15

(1/2)x +1 =  -3x +15, add 3x and subtract1

(1/2) x+3x = 15-1, combine like terms

(7/2)x = 14 , multiply both sides by 2

7x = 28, divide both sides by 7

x= 4

y= (1/2)x +1 = (4/2) +1 =3 so R(4,3)

- the area of ΔPQR is (base *height)/2

base= 15-1= 14

height = 4

A= (14*4)/2 = 14*2 = 28

Which of the following is the point and slope of the equation y + 14 = 7(x - 18)?

Answers

Answer:

y = 7x - 140

The slope is 7

The y-intersept is -140

= (7, -140)

Step-by-step explanation:

y + 14 = 7(x - 18)

y + 14 = 7x - 126

y =7x - 126 - 14

y = 7x - 140

Find the range from the ordered pair {(1, 2), (2, 3), (3, 4), (4, 5)}

Answers

Answer:

Range { 2,3,4,5}

Step-by-step explanation:

The range is the output values

Range { 2,3,4,5}

57 117find x triangle ​

Answers

Answer:

60

Step-by-step explanation:

x = 180 - [ 57 + ( 180 - 117 ) ]

= 180 - [ 57 + 63 ]

= 180 - 120

x = 60

What is the solution to the inequality -6+|2p+3| > 7

Answers

Step-by-step explanation:

you're going to have to set up two expressions since it's an absolute value problem

HELPPP PLEASE ASAP!!! I don’t know how to solve this problem nor where to start? Can some please help me out and explain how you got the answer please. Thank you for your time.

Answers

Step-by-step explanation:

Count the number of times you have to move the decimal point to the right until it is to the right of the 1st nonzero number.

a) You have to move the decimal point 11 times until it gets to the right of the 1st nonzero number, which is 7. You then rewrite this number as

[tex]7.2×10^{-11}[/tex]

The exponent of 10 is a negative number because you moved the decimal point to the right.

b) Similarly, you have to move the point 9 times to the right so the answer is

[tex]9.5×10^{-9}[/tex]

Can someone please help me?

Answers

you have to read the bottom link for the answer key

HELPPPP

what is this

Answers

Answer:

Pls be specific with your question

Find the unlabeled side length

Answers

Answer:

hope it helps you.......

Answer:

The unidentified length is 13

Step-by-step explanation:

To solve this we have to use the Pythagorean Theorem

[tex]a^2+b^2=c^2\\5^2+12^2=c^2\\25+144=c^2\\169=c^2\\13=c[/tex]

Yanni read 24 pages of
a book. 1 of the book is
still left to read. How many
pages are there in the
whole book?

Answers

There are 23 pages left to read

79
Work out the circumference of this circle.
Take a to be 3.142 and write down all the digits given by your calculator.
14 cm

Answers

Answer: 43.988

Step-by-step explanation: The formula for the circumference of a circle is the diameter multiplied by pi. Since the diameter is 14 and it is telling us to use 3.142 as pi, we can multiply the two and get the answer.

Other Questions
An automobile manufacturing plant produced vehicles today: were sedans, were trucks, and were motorcycles. Plant managers are going to select two of these vehicles for a thorough inspection. The first vehicle will be selected at random, and then the second vehicle will be selected at random from the remaining vehicles. What is the probability that two motorcycles will be selected What was the situation before the development? Which investment is best for someone who is likely to need cash soon? The following relates to a proposed equipment purchase: Cost $ 157,000 Salvage value $ 5,000 Estimated useful life 4 years Annual net cash flows $ 52,600 Depreciation method Straight-line Ignoring income taxes, the annual net income amount used to calculate the accounting rate of return is 3 1/2 1 1/4khan academy answer in simplified proper fractionorsimplified improper fraction Rosie is an employee of Social Media Company. At the end of a successful unionization campaign, Tech Workers Union is certified as the employees bargaining representative. Rosie refuses to join the union. For this refusal, under the National Labor Relations Act, the union can force the company to PLS HELP!!! The problem is in the photo attached Can someone help me with this math homework please! A biased 3-coloured spinner was spun 240 times. It landed on Red n times, on Orange 3n times and on Yellow 8n times. If you spin this spinner 1000 times, how many times do you expect it to land on Red? plz help me u will give u like and follow u back plz help me Kasey has three pieces of wood that measure 9 inches, 12 inches, and 21 inches. Can she make a right trianglewith the three pieces of wood (without cutting or overlapping)? Find the distance between the two points rounding to the nearest tenth (if necessary). ( 8 , 6 ) and ( 6 , 0 ) How was industrialization in Great Britain different from industrialization in China. I need help with this 1 use the the dot strict on the right to and the periodic table to determine the element represented. sodium (Na)potassium (K)neon (Ne)argon (Ar) Please help me find this box guys Im struggling Which expression is equivalent to 2(a+2b)-a-2bo 3a+2bo 3a-2bo a-2b You are considering purchasing a bond with 18 years to maturity, a $1,000 face value and a 5% coupon rate with interest paid semiannually. Interest rates on bonds in this risk class have dropped to 3% and you expect that this bond will be called in 3 years. The current price of the bond is $1,075.00. Assuming you would be paid one year's worth of interest as a premium if the bond is called, what is the yield to call on this bond?a. 5.43%.b. 3.91%.c. 0 2.93%.d. 1.86%. The speed of a toy car is 20 m/s. The time taken for a trip is 200seconds. Find thedistance covered in meter Which of the following words is generally used to describe what managers do as opposed to what leaders do? a) influence b) organize c) inspire innovate d)